You are on page 1of 6

MATH 141 Sample Final Exam Solutions

Page 1 of 6

April 2014

Part A: MULTIPLE CHOICE QUESTIONS 01 b 02 b 03 c 04 e 05 d 06 c 07 d 08 c 09 e 10 d 11 a 12 a

Part B: FULL ANSWER QUESTIONS 1. Let R be the region in the rst quadrant bounded by the x-axis, the y -axis, the line y = ln 2 and the graph of y = ln x. Let S be the solid obtained by revolving R about the y -axis. (a) Compute the volume of S with the disc/washer method. (b) Compute the volume of S with the method of cylindrical shells. Solution:
0.8 0.6 0.4 0.2

0.5 0.2

1.0

1.5

2.0

(a) For the disc method, the representative rectangles have to be perpendicular to the axis of revolution i.e. the y -axis. The representative rectangles are thus horizontal i.e. we integrate with respect to y . The region R is just the area under the graph of x = ey for y between 0 and ln 2. Thus [ ] ln 2 ln 2 1 2y ln 2 3 y 2 2y Vol(S ) = (e ) dy = e dy = e = (4 1) = 2 2 2 0 0 0 (b) For the shell method, the representative rectangles have to be parallel to the axis of revolution i.e. the y -axis. The representative rectangles are thus vertical i.e. we integrate with respect to x. The region R has to be split along the line x = 1 as indicated in the sketch above. Thus 1 2 Vol(S ) = 2 x ln 2 dx + 2 x(ln 2 ln x) dx 0 1 2 2 1 x ln x dx = 2 x ln 2 dx + 2 x ln 2 dx 2 1 1 0 [ ] 2 2 2 1 2 2 = 2 x ln 2 dx 2 x ln x dx = 2 ln 2 x 2 x ln x dx 2 0 1 1 0 2 = 4 ln 2 2 x ln x dx
1

Using integration by parts yields ([ ) ( ]2 [ ] ) 1 2 1 2 2 1 1 1 2 2 = 4 ln 2 2 x ln x x dx = 4 ln 2 2 2 ln 2 x 2 2 1 x 2 2 1 1 3 3 = 4 ln 2 2 (2 ln 2 ) = 4 2

MATH 141 Sample Final Exam Solutions

Page 2 of 6

April 2014

2. Compute the following integrals: )3 ( x (a) dx 3 x2 Solution:

(b)

2 x dx Hint: Start with the subst. u = x. 2 x 1

(a) We use the trigonometric substitution x = 3 sin , dx = 3 cos d, 3 x2 = 3 cos . We get: )3 )3 ( ( sin3 x 3 sin dx = 3 cos d = 3 d cos2 3 cos 3 x2 Since the power of sine is odd we use the substitution u = cos , du = sin d sin2 1 cos2 1 u2 = 3 sin d = 3 sin d = 3 du cos2 cos2 u2 ( ) ( ) 1 1 1 = 3 1 du = 3 u + C = 3 +u +C u2 u u ( ) 3 x2 3 = 3(cos + sec ) + C = 3 +C + 3 3 x2 3 = 3 x2 + +C 3 x2 1 (b) With the substitution u = x, du = 2 dx, 2 x du = dx, 2u du = dx we obtain: x 2u 4u2 4u2 4u2 2 x dx = 2 u du = du = du = du 2 4 4 2 2 x 1 u 1 u 1 (u 1)(u + 1) (u 1)(u + 1)(u2 + 1) We determine the partial fraction decomposition: a b cu + d 4u2 = + + 2 2 (u 1)(u + 1)(u + 1) u1 u+1 u +1 or a(u + 1)(u2 + 1) + b(u 1)(u2 + 1) + (cu + d)(u2 1) = 4u2 We plug in various values for u: u=1: u = 1 : u=0: u=2: Thus 4a = 4, a = 1 4b = 4, b = 1 a b d = 1 (1) d = 2 d = 0, d = 2 15a + 5b + (2c + d)3 = 15 5 + 6c + 6 = 16 + 6c = 16, c = 0 4u2 1 1 2 = + 2 2 (u 1)(u + 1)(u + 1) u1 u+1 u +1 4u2 du = ln |u 1| ln |u + 1| + 2 arctan u + C (u 1)(u + 1)(u2 + 1) 2 x dx = ln | x 1| ln | x + 1| + 2 arctan x + C 2 x 1

and

Finally,

MATH 141 Sample Final Exam Solutions

Page 3 of 6

April 2014

3. Determine for each of the following improper integrals whether it converges or diverges. Compute the values of the convergent integrals. (a)
0 1/e

1 dx x ln3 x

(b)

1/e

1 dx x ln3 x

(c)
0

1 dx x ln3 x

Solution: We compute

1 1 dx by using the substitution u = ln x, du = x dx: x ln3 x 1 1 1 1 1 du = u2 + C = +C 3 dx = 3 u 2 2 ln2 x x ln x 1 dx = lim t0+ x ln3 x 1 dx = lim t1 x ln3 x
t 1/e

(a)

1/e

[ ] 1 1 1 1/e 1 1 1 lim = lim [1 2 ] = 3 dx = 2 t 2 + + 2 t0 2 0 x ln x ln x t ln t

(b)

1/e

1/e

[ ] 1 1 1 t 1 1 lim = lim [ 2 1] = 3 dx = 2 t 2 2 t1 ln t 1 ln x 1/e x ln x

i.e. the improper integral diverges. (c) This integral is improper at both limits of integration. It converges if and only if both of the integrals 1 1 of parts (a) and (b) converge. Since (b) diverges, 3 dx diverges. 0 x ln x

MATH 141 Sample Final Exam Solutions

Page 4 of 6

April 2014

4. Consider the parametric curve dened by the equations x(t) = sin2 t y (t) = sin t cos t for all t with 0 t /2. (a) Find the area between the curve and the x-axis. (b) Find the surface area of the surface of revolution obtained by revolving this curve about the x-axis. Solution: Remark: This curve can be shown to be a semicircle of radius 1/2. The solution below will not make use of this. (a) A=
0 /2

y (t)x (t) dt =
0 /2

/2

(sin t cos t)(2 sin t cos t) dt = 2


0 /2

/2

sin2 cos2 t dt

1 1 1 /2 1 1 2 (1 cos(2t)) (1 + cos(2t) dt = 1 cos (2t) dt = 1 (1 + cos(4t)) dt =2 2 2 2 2 2 0 0 0 [ ]/2 /2 /2 ] 1 1 1 1 1 1 1[ = cos(4t) dt = 1 cos(4t) dt = t sin(4t) = ( 0) 0 2 0 2 2 4 0 4 4 4 2 0 = 8 (b) S = 2 = 2 = 2 = 2
0 0 /2 0 /2 0 /2 /2

2 2 y (t) x (t) + y (t) dt = 2 sin t cos t sin t cos t


0

/2

sin t cos t (2 sin t cos t)2 + (cos2 t sin2 t)2 dt

4 sin2 t cos2 t + cos4 t 2 sin2 t cos2 t + sin4 t dt cos4 t + 2 sin [


2 /2

sin t cos t (sin2 t + cos2 t)2 dt

t cos2 t ]/2

+ sin t dt = 2
0

1 sin t cos t dt = 2 sin2 t 2

=
0

MATH 141 Sample Final Exam Solutions

Page 5 of 6

April 2014

5. Determine whether the following series converge or diverge. Fully justify your answers. (a)
n=1

cos(1/n)

(b)

3n + 5 2 n5 1 n=1

(c)

n=2

1 n ln n

(d)

(1)n+1

n=1

e1/n n

Solution: (a) lim an = lim cos(1/n) = cos(0) = 1. Since this limit does not equal zero, the series diverges by the n n nth term divergence test. n 1 = (b) We use the limit comparison test and compare with the (convergent) P -series : 3 n5 n=1 n /2 n=1 an 3n + 5 3n5/2 + 5n3/2 = lim n3/2 = lim n bn n n 2 n5 1 2n5 1 lim Dividing numerator and denominator by the highest power of n i.e. n5/2 yields
5 3+ n 3 = lim = < n 1 2 2 n 5 3n + 5 converges by the limit comparison test. Thus 2n5 1 n=1

(c) We use the integral test. Let f (x) =

1 . Then f (x) is positive for x > 1 and decreasing (since the x ln x 1 denominator is increasing). Using the substitution u = ln x, du = x dx we get 1 1 dx = du = ln |u| + C = ln | ln(x)| + C x ln x u
2

and 1 dx = lim = b x ln x
2 b

1 dx = lim (ln | ln b| ln | ln 2|) = lim (ln(ln b) ln(ln 2)) = b b x ln x


n=2

Thus the integral diverges and, by the integral test, the series

1 diverges. n ln n

(d) We use the alternating series test. e1/n e1/n lim e1/n = e0 = 1. Thus lim = 0. We also have to show that the sequence an = is decreasing. n n n n e1/x Let f (x) = . Then x ( 1) (1 ) x e1/x x 1 e1/x e1/x x +1 2 f (x) = = <0 x2 x2 and f (x) and thus an are decreasing. The series series test.

(1)n+1

n=1

e1/n thus converges by the alternating n

MATH 141 Sample Final Exam Solutions

Page 6 of 6

April 2014

6. Compute the perimeter (or total arc length of the boundary) of the region outside the cardioid r = 1 + sin and inside the circle r = cos (see gure below). Solution: The boundary of the region consists of two curves. The lower curve is just a semi-circle of radius 1/2 with arc length L1 = 1 2 2r = 2 . The upper curve is the part of the cardioid between = /2 and = 0 (the circle and the cardioid intersect at = /2 with radius 0 and at = 0 with radius 1). The arc length L2 of the upper curve is thus ( )2 0 0 dr r2 + d = (1 + sin )2 + cos2 d L2 = d /2 /2 0 0 = 1 + 2 sin + sin2 + cos2 d = 2 + 2 sin d = 2 = 2 = 2
/2 0 /2

1 + sin d

/2 0 /2

0 0 1 + sin 1 sin 1 sin2 cos2 d = 2 d = 2 d 1 sin 1 sin 1 sin /2 /2 0 | cos | cos d = 2 d 1 sin 1 sin /2

/2

Using the substitution u = 1 sin , du = cos d, du = cos d yields = 2


2 1

1 du = 2 u

[ ]2 1 du = 2 2 u 1 = 2 2( 2 1) = 4 2 2 u

The perimeter is thus L = L1 + L2 =

+ 4 2 2. 2

You might also like